Evolve Fundamentals HESI

Réussis tes devoirs et examens dès maintenant avec Quizwiz!

A client who is dying jokes about the situation even though the client is becoming sicker and weaker. Which is the most therapeutic response by the nurse?

"Does it help to joke about your illness?"

The registered nurse is teaching a nursing student about providing care to an older adult with dementia. Which statement made by the nursing student indicates a need for further education?

"I should monitor weight and food intake once in a month."

A nurse hired to work in a metropolitan hospital provides services for a culturally diverse population. One of the nurses on the unit says it is the nurses' responsibility to discourage "these people" from bringing all that "alternative medicine stuff" to their family members. Which response by the recently hired nurse is most appropriate?

"Nontraditional approaches to health care can be beneficial."

An 80-year-old client is admitted to the hospital because of complications associated with severe dehydration. The client's daughter asks the nurse how her mother could have become dehydrated, because she is alert and able to care for herself. The nurse's best response is:

"The thirst reflex diminishes with age, and therefore the recognition of the need for fluid is decreased."

A newly hired nurse during orientation is approached by a surveyor from the department of health. The surveyor asks the nurse about the best way to prevent the spread of infection. What is the most appropriate nursing response?

"Wash your hands before and after any client care."

A client is dying. Hesitatingly, his wife says to the nurse, "I'd like to tell him how much I love him, but I don't want to upset him." Which is the best response by the nurse?

"You should share your feelings with him while you can."

A nurse teaches a client about wearing thigh-high antiembolism elastic stockings. What would be appropriate to include in the instructions?

"You will need to apply them in the morning before you lower your legs from the bed to the floor."

Heparin 20,000 units in 500 ml D5W at 50 ml/hour has been infusing for 5½ hours. How much heparin has the client received? A. 11,000 units. B. 13,000 units. C. 15,000 units. D. 17,000 units.

(A) is the correct calculation: 20,000 units/500 ml = 40 units (the amount of units in one ml of fluid). 40 units/ml x 50 ml/hr = 2,000 units/hour (1,000 units in 1/2 hour). 5.5 x 2,000 = 11,000 (A). OR, multiply 5 x 2,000 and add the 1/2 hour amount of 1,000 to reach the same conclusion = 11,000 units. Correct Answer: A

The healthcare provider prescribes furosemide (Lasix) 15 mg IV stat. On hand is Lasix 20 mg/2 ml. How many milliliters should the nurse administer? A. 1 ml. B. 1.5 ml. C. 1.75 ml. D. 2 ml.

(B) is the correct calculation: Dosage on hand/amount on hand = Dosage desired/x amount. 20 mg : 2 ml = 15 mg : x . 20x = 30. x = 30/20; = 1½ or 1.5 ml. Correct Answer: B

At the time of the first dressing change, the client refuses to look at her mastectomy incision. The nurse tells the client that the incision is healing well, but the client refuses to talk about it. What would be an appropriate response to this client's silence? A. "It is normal to feel angry and depressed, but the sooner you deal with this surgery, the better you will feel." B. "Looking at your incision can be frightening, but facing this fear is a necessary part of your recovery." C. "It is OK if you don't want to talk about your surgery. I will be available when you are ready." D. "I will ask a woman who has had a mastectomy to come by and share her experiences with you."

(C) displays sensitivity and understanding without judging the client. (A) is judgmental in that it is telling the client how she feels and is also insensitive. (B) would give the client a chance to talk, but is also demanding and demeaning. (D) displays a positive action, but, because the nurse's personal support is not offered, this response could be interpreted as dismissing the client and avoiding the problem. Correct Answer: C

The healthcare provider prescribes the diuretic metolazone (Zaroxolyn) 7.5 mg PO. Zaroxolyn is available in 5 mg tablets. How much should the nurse plan to administer? A. ½ tablet. B. 1 tablet. C. 1½ tablets. D. 2 tablets.

(C) is the correct calculation: D/H × Q = 7.5/5 × 1 tablet = 1½ tablets. Correct Answer: C

Twenty minutes after beginning a heat application, the client states that the heating pad no longer feels warm enough. What is the best response by the nurse? A. "That means you have derived the maximum benefit, and the heat can be removed." B. "Your blood vessels are becoming dilated and removing the heat from the site." C. "We will increase the temperature 5 degrees when the pad no longer feels warm." D. "The body's receptors adapt over time as they are exposed to heat."

(D) describes thermal adaptation, which occurs 20 to 30 minutes after heat application. (A and B) provide false information. (C) is not based on a knowledge of physiology and is an unsafe action that may harm the client. Correct Answer: D

An IV infusion terbutaline sulfate 5 mg in 500 ml of D5W, is infusing at a rate of 30 mcg/min prescribed for a client in premature labor. How many ml/hr should the nurse set the infusion pump? A. 30 B. 60 C. 120 D. 180

(D) is correct calculation: 180 ml/hr = 500 ml/5 mg × 1mg/1000 mcg × 30 mcg/min × 60 min/hr. Correct Answer: D

The nurse mixes 50 mg of Nipride in 250 ml of D5W and plans to administer the solution at a rate of 5 mcg/kg/min to a client weighing 182 pounds. Using a drip factor of 60 gtt/ml, how many drops per minute should the client receive? A. 31 gtt/min. B. 62 gtt/min. C. 93 gtt/min. D. 124 gtt/min.

(D) is the correct calculation: Convert lbs to kg: 182/2.2 = 82.73 kg. Determine the dosage for this client: 5 mcg × 82.73 = 413.65 mcg/min. Determine how many mcg are contained in 1 ml: 250/50,000 mcg = 200 mcg per ml. The client is to receive 413.65 mcg/min, and there are 200 mcg/ml; so the client is to receive 2.07ml per minute. With a drip factor of 60 gtt/ml, then 60 × 2.07 = 124.28 gtt/min (D) OR, using dimensional analysis: gtt/min = 60 gtt/ml X 250 ml/50 mg X 1 mg/1,000 mcg X 5 mcg/kg/min X 1 kg/2.2 lbs X 182 lbs. Correct Answer: D

The UAPs working on a chronic neuro unit ask the nurse to help them determine the safest way to transfer an elderly client with left-sided weakness from the bed to the chair. What method describes the correct transfer procedure for this client? A. Place the chair at a right angle to the bed on the client's left side before moving. B. Assist the client to a standing position, then place the right hand on the armrest. C. Have the client place the left foot next to the chair and pivot to the left before sitting. D. Move the chair parallel to the right side of the bed, and stand the client on the right foot.

(D) uses the client's stronger side, the right side, for weight-bearing during the transfer, and is the safest approach to take. (A, B, and C) are unsafe methods of transfer and include the use of poor body mechanics by the caregiver. Correct Answer: D

The healthcare provider prescribes morphine sulfate 4mg IM STAT. Morphine comes in 8 mg per ml. How many ml should the nurse administer?

0.5 ml.

Seconal 0.1 gram PRN at bedtime is prescribed to a client for rest. The scored tablets are labeled grain 1.5 per tablet. How many tablets should the nurse plan to administer?

1 tab

Secobarbital (Seconal) 150 mg is prescribed at bedtime for a male client who is scheduled for surgery in the morning. The scored tablets are labeled 0.1 gram/tablet. How many tablets should the nurse administer? (Enter numerical value only. If rounding is required, round to the nearest tenth.)

1,000 mg : 1 gram :: X mg : 0.1 gram X = 100 mg D/H = 150/100 = 1.5 tablets Correct Answer: 1.5

The healthcare provider prescribes furosemide (Lasix) 15 mg IV stat. On hand is Lasix 20 mg/2 ml. How many milliliters should the nurse administer?

1.5ml

Heparin 20,000 units in 500 ml D5W at 50 ml/hour has been infusing for 5½ hours. How much heparin has the client received?

11,000 units

The nurse mixes 50 mg of Nipride in 250 ml of D5W and plans to administer the solution at a rate of 5 mcg/kg/min to a client weighing 182 pounds. Using a drip factor of 60 gtt/ml, how many drops per minute should the client receive?

124 gtt/min.

A client is to receive cimetidine (Tagamet) 300 mg q6h IVPB. The preparation arrives from the pharmacy diluted in 50 ml of 0.9% NaCl. The nurse plans to administer the IVPB dose over 20 minutes. For how many ml/hr should the infusion pump be set to deliver the secondary infusion?

150

An IV infusion terbutaline sulfate 5 mg in 500 ml of D5W, is infusing at a rate of 30 mcg/min prescribed for a client in premature labor. How many ml/hr should the nurse set the infusion pump?

180

The healthcare provider prescribes the diuretic metolazone (Zaroxolyn) 7.5 mg PO. Zaroxolyn is available in 5 mg tablets. How much should the nurse plan to

1½ tablets

The nurse prepares a 1,000 ml IV of 5% dextrose and water to be infused over 8 hours. The infusion set delivers 10 drops per milliliter. The nurse should regulate the IV to administer approximately how many drops per minute?

21

The healthcare provider prescribes 1,000 ml of Ringer's Lactate with 30 Units of Pitocin to run in over 4 hours for a client who has just delivered a 10 pound infant by cesarean section. The tubing has been changed to a 20 gtt/ml administration set. The nurse plans to set the flow rate at how many gtt/min?

83 gtt/min.

A client is receiving a cephalosporin antibiotic IV and complains of pain and irritation at the infusion site. The nurse observes erythema, swelling, and a red streak along the vessel above the IV access site. Which action should the nurse take at this time? A. Administer the medication more rapidly using the same IV site. B. Initiate an alternate site for the IV infusion of the medication. C. Notify the healthcare provider before administering the next dose. D. Give the client a PRN dose of aspirin while the medication infuses.

A cephalosporin antibiotic that is administered IV may cause vessel irritation. Rotating the infusion site minimizes the risk of thrombophlebitis, so an alternate infusion site should be initiated (B) before administering the next dose. Rapid administration (A) of intravenous cephalosporins can potentiate vessel irritation and increase the risk of thrombophlebitis. (C) is not necessary to initiate an alternative IV site. Although aspirin has antiinflammatory actions, (D) is not indicated. Correct Answer: B

A female client asks the nurse to find someone who can translate into her native language her concerns about a treatment. Which action should the nurse take? A. Explain that anyone who speaks her language can answer her questions. B. Provide a translator only in an emergency situation. C. Ask a family member or friend of the client to translate. D. Request and document the name of the certified translator.

A certified translator should be requested to ensure the exchanged information is reliable and unaltered. To adhere to legal requirements in some states, the name of the translator should be documented (D). Client information that is translated is private and protected under HIPAA rules, so (A) is not the best action. Although an emergency situation may require extenuating circumstances (B), a translator should be provided in most situations. Family members may skew information and not translate the exact information, so (C) is not preferred. Correct Answer: D

Which description is most appropriate for the family centered care approach?

A collaborative plan of care is developed to achieve optimal health.

What is the most important reason for starting intravenous infusions in the upper extremities rather than the lower extremities of adults?

A decreased flow rate could result in the formation of a thrombosis.

The nurse is using a genogram while conducting a client's health assessment and past medical history. What information should the genogram provide? A. Genetic and familial health disorders. B. Chronic health problems. C. Reason for seeking health care. D. Undetected disorders.

A genogram that is used during the health assessment process identifies genetic and familial health disorders (A). It may not identify the client's chronic health problems (B), so it is not a reason to seek health care (C). A genogram is not a diagnostic tool detect disorders (D), such as those based on pathological findings or DNA. Correct Answer: A

The nurse is assessing the nutritional status of several clients. Which client has the greatest nutritional need for additional intake of protein? A. A college-age track runner with a sprained ankle. B. A lactating woman nursing her 3-day-old infant. C. A school-aged child with Type 2 diabetes. D. An elderly man being treated for a peptic ulcer.

A lactating woman (B) has the greatest need for additional protein intake. (A, C, and D) are all conditions that require protein, but do not have the increased metabolic protein demands of lactation. Correct Answer: B

The nurse is assessing the nutritional status of several clients. Which client has the greatest nutritional need for additional intake of protein?

A lactating woman nursing her 3-day-old infant.

A client has been diagnosed with type 1 diabetes mellitus. When providing instructions on sharps disposal, the nurse should instruct the client to place the syringes in what?

A plastic liquid detergent bottle with a screw-top lid

The nurse plans to obtain health assessment information from a primary source. Which option is a primary source for the completion of the health assessment? A. Client. B. Healthcare provider. C. A family member. D. Previous medical records.

A primary source of information for a health assessment is the client (A). (B, C, and D) are considered secondary sources about the client's health history, but other details, such as subjective data, can only be provided directly form the client. Correct Answer: A

A postoperative client will need to perform daily dressing changes after discharge. Which outcome statement best demonstrates the client's readiness to manage his wound care after discharge? The client A. asks relevant questions regarding the dressing change. B. states he will be able to complete the wound care regimen. C. demonstrates the wound care procedure correctly. D. has all the necessary supplies for wound care.

A return demonstration of a procedure (C) provides an objective assessment of the client's ability to perform a task, while (A and B) are subjective measures. (D) is important, but is less of a priority prior to discharge than the nurse's assessment of the client's ability to complete the wound care. Correct Answer: C

A client with chronic kidney disease selects scrambled egg for his breakfast. What action should the nurse take? A) Commend the client for selecting a high biologic value protein. B) Remind the client that protein in the diet should be avoided. C) Suggest that the client also select orange juice, to promote absorption. D) Encourage the client to attend classes on dietary management of CKD.

A) Commend the client for selecting a high biologic value protein.

The nurse observes that a male client has removed the covering from an ice pack applied to his knee. What action should the nurse take first? A) Observe the appearance of the skin under the ice pack B) Instruct the client regarding the need for the covering. C) Reapply the covering after filling with fresh ice. D) Ask the client how long the ice was applied to the skin.

A) Observe the appearance of the skin under the ice pack.

During shift change report, the nurse receives report that a client has abnormal heart sounds. Which placement of the stethoscope should the nurse use to hear the client's hear sounds? A. Place the stethoscope bell at random points on the posterior chest. B. Use the stethoscope bell over the valvular areas of the anterior chest. C. Move the diaphragm of the stethoscope over the left anterior chest, D. Position the diaphragm of the stethoscope at Erb's point on the chest.

Abdominal heart sounds are best heard with the bell of the stethoscope, which picks up lower-pitched sounds, that is placed at points on the anterior chest (B). (A, C, and D) do not provide the best assessment of abdominal heart valve sounds. Correct Answer: B

Which critical thinking skill does the nurse associate with the concept of maturity?

Ability to reflect on own judgments

Which response by a client with a nursing diagnosis of "Spiritual distress," indicates to the nurse that a desired outcome measure has been met? A. Express concern about the meaning and importance of life. B. Remains angry at God for the continuation of the illness. C. Accepts that punishment from God is not related to illness. D. Refuses to participate in religious rituals that have no meaning.

Acceptance that she is not being punished by God indicates a desired outcome (C) for some degree of resolution of spiritual distress. (A, B, and D) do not support the concept of grief, loss, and cultural/spiritual acceptance. Correct Answer: C

When planning discharge teaching for a young adult, the nurse should include the potential health problems common in this age group. What should the nurse include in this teaching plan?

Accidents, including their prevention

While instructing a male client's wife in the performance of passive range-of-motion exercises to his contracted shoulder, the nurse observes that she is holding his arm above and below the elbow. What nursing action should the nurse implement?

Acknowledge that she is supporting the arm correctly.

On the third postoperative day after a below-the-knee amputation, a client is refusing to eat, talk, or perform any rehabilitative activities. What is the best initial approach that the nurse should take when interacting with this client?

Acknowledge that the client's withdrawal is an expected and necessary part of initial grieving.

Which nursing action is most important when caring for a client using medications to manage disease in the hospital?

Administering the medications

A hospitalized male client is receiving nasogastric tube feedings via a small-bore tube and a continuous pump infusion. He reports that he had a bad bout of severe coughing a few minutes ago, but feels fine now. What action is best for the nurse to take?

After clearing the tube with 30 ml of air, check the pH of fluid withdrawn from the tube.

What action should the nurse implement when accessing an implemented infusion port for a client who receives long term IV medications? A. Cleanse the site with iodine solution. B. insert a Huber-point needle into the port. C. Flush tubing with 5 mL of normal saline. D. Place a sterile dressing over the port.

An implanted infusion port needs to be accessed using a Huber-point needle (B) (non-coring) to prevent damage to the self-sealing septum of the port. (A, C, and D) are not necessary when accessing an implanted infusion port. Corrected Answer: B

A client with pneumonia has a decrease in oxygen saturation from 94% to 88% while ambulating. Based on these findings, which intervention should the nurse implement first? A. Assist the ambulating client back to the bed. B. Encourage the client to ambulate to resolve pneumonia. C. Obtain a prescription for portable oxygen while ambulating. D. Move the oximetry probe from the finger to the earlobe.

An oxygen saturation below 90% indicates inadequate oxygenation. First, the client should be assisted to return to bed (A) to minimize oxygen demands. Ambulation increases aeration of the lungs to prevent pooling of respiratory secretions, but the client's activity at this time is depleting oxygen saturation of the blood, so (B) is contraindicated. Increased activity increases respiratory effort, and oxygen may be necessary to continue ambulation (C), but first the client should return to bed to rest. Oxygen saturation levels at different sites should be evaluated after the client returns to bed (D). Correct Answer: A

During a routine checkup a client reports concerns over weight gain despite trying juice cleanses and other trend diets. The nurse records the client's weight and BMI at a healthy range, but the client states, "I wish I were as thin as my co-workers." The client is at risk for what culturally-bound condition?

Anorexia nervosa

The nurse is caring for a client admitted with chronic obstructive pulmonary disease (COPD). The nurse should monitor the results of which laboratory test to evaluate the client for hypoxia?

Arterial blood gas

Which intervention is most important for the nurse to implement for a male client who is experiencing urinary retention?

Assess for bladder distention.

A client with pneumonia has a decrease in oxygen saturation from 94% to 88% while ambulating. Based on these findings, which intervention should the nurse implement first?

Assist the ambulating client back to the bed.

Which statement is true for attachment in the newborn?

Attachment is the interaction between parent and child.

A nurse in the ambulatory preoperative unit identifies that a client is more anxious than most clients. What is the nurse's best intervention?

Attempt to identify the client's concerns.

The nurse is teaching a client with numerous allergies how to avoid allergens. Which instruction should be included in this teaching plan?

Avoid any types of sprays, powders, and perfumes.

Three days following surgery, a male client observes his colostomy for the first time. He becomes quite upset and tells the nurse that it is much bigger than he expected. What is the best response by the nurse? A) Reassure the client that he will become accustomed to the stoma appearance in time. B) Instruct the client that the stoma will become smaller when the initial swelling diminishes. C) Offer to contact a member of the local ostomy support group to help him with his concerns. D) Encourage the client to handle the stoma equipment to gain confidence with the procedure.

B) Instruct the client that the stoma will become smaller when the initial swelling diminishes.

A female client with a nasogastric tube attached to low suction states that she is nauseated. The nurse assesses that there has been no drainage through the nasogastric tube in the last two hours. What action should the nurse take first? A) Irrigate the nasogastric tube with sterile normal saline. B) Reposition the client on her side. C) Advance the nasogastric tube an additional five centimeters. D) Administer an intravenous antiemetic prescribed for PRN use.

B) Reposition the client on her side.

In developing a plan of care for a client with dementia, the nurse should remember that confusion in the elderly A) is to be expected, and progresses with age B) often follows relocation to new surroundings C) is a result of irreversible brain pathology D) can be prevented with adequate sleep

B) often follows relocation to new surroundings

An obese male client discusses with the nurse his plans to begin a long-term weight loss regimen. In addition to dietary changes, he plans to begin an intensive aerobic exercise program 3 to 4 times a week and to take stress management classes. After praising the client for his decision, which instruction is most important for the nurse to provide?

Be sure to have a complete physical examination before beginning your planned exercise program.

After completing an assessment and determining that a client has a problem, which action should the nurse perform next? A. Determine the etiology of the problem. B. Prioritize nursing care interventions. C. Plan appropriate interventions. D. Collaborate with the client to set goals.

Before planning care, the nurse should determine the etiology, or cause, of the problem (A), because this will help determine (B, C, and D). Correct Answer: A

A client who is a Jehovah's Witness is admitted to the nursing unit. Which concern should the nurse have for planning care in terms of the client's beliefs? A. Autopsy of the body is prohibited. B. Blood transfusions are forbidden. C. Alcohol use in any form is not allowed. D. A vegetarian diet must be followed.

Blood transfusions are forbidden (B) in the Jehovah's Witness religion. Judaism prohibits (A). Buddhism forbids the use of (C) and drugs. Many of these sects are vegetarian (D), but the direct impact on nursing care is (B). Correct Answer: B

A client who is a Jehovah's Witness is admitted to the nursing unit. Which concern should the nurse have for planning care in terms of the client's beliefs?

Blood transfusions are forbidden.

Which assessment data would provide the most accurate determination of proper placement of a nasogastric tube? A. Aspirating gastric contents to assure a pH value of 4 or less. B. Hearing air pass in the stomach after injecting air into the tubing. C. Examining a chest x-ray obtained after the tubing was inserted. D. Checking the remaining length of tubing to ensure that the correct length was inserted.

Both (A and B) are methods used to determine proper placement of the NG tubing. However, the best indicator that the tubing is properly placed is (C). (D) is not an indicator of proper placement. Correct Answer: C

Which activity would the nurse use as an example of fine motor skills of infants aged 2 to 4 months?

Bringing objects from hand to mouth

A hospitalized male client is receiving nasogastric tube feedings via a small-bore tube and a continuous pump infusions. He reports that he had a bad bout of severe coughing a few minutes ago, but feels fine now. What action is best for the nurse to take? A) Record the coughing incident. No further action is required at this time. B) Stop the feeding, explain to the family why it is being stopped and notify the healthcare provider. C) After clearing the tube with 30 mL of air, check the pH of fluid withdrawn from the tube. D) Inject 30 mL of air into the tube while auscultating the epigastrium for gurgling.

C) After clearing the tube with 30 ml of air, check the pH of fluid withdrawn from the tube.

Which assessment data provides the most accurate determination of proper placement of a nasogastric tube? A) Aspirating gastric contents to assure a pH value of 4 or less. B) Hearing air pass in the stomach after injecting air into the tubing. C) Examining a chest x-ray obtained after the tubing was inserted. D) Checking the remaining length of tubing to ensure that the correct length was inserted.

C) Examining a chest x-ray obtained after the tubing was inserted.

A postoperative client will need to perform daily dressing changes after discharge. Which outcome statement best demonstrates the client's readiness to manage his wound care after discharge? The client A) asks relevant questions regarding the dressing change B) states he will be able to complete the wound care regimen C) demonstrates the wound care procedure correctly D) has all the necessary supplies for wound care.

C) demonstrates the wound care procedure correctly

A young mother of three children complains of increased anxiety during her annual physical exam. What information should the nurse obtain first? A. Sexual activity patterns. B. Nutritional history. C. Leisure activities. D. Financial stressors.

Caffeine, sugars, and alcohol can lead to increased levels of anxiety, so a nutritional history (C) should be obtained first so that health teaching can be initiated if indicated. (A and C) can be used for stress management. Though (D) can be a source of anxiety, a nutritional history should be obtained first. Correct Answer: B

The nurse is caring for a client before, during, and immediately after surgery. Which type of care is provided to the client?

Care that supports homeostatic regulation

An advanced practice registered nurse (APRN) is caring for a pregnant woman. Which type of APRN would care for this client?

Certified nurse midwife (CNM)

On admission, a client presents a signed living will that includes a Do Not Resuscitate (DNR) prescription. When the client stops breathing, the nurse performs cardiopulmonary resuscitation (CPR) and successfully revives the client. What legal issues could be brought against the nurse? A. Assault. B. Battery. C. Malpractice. D. False imprisonment.

Civil laws protect individual rights and include intentional torts, such as assault (an intentional threat to engage in harmful contact with another) or battery (unwanted touching). Performing any procedure against the client's wishes can potentially pose a legal issue, such as battery (B), even if the procedure is of questionable benefit to the client. (A, C, and D) are not examples against the client's request. Correct Answer: B

How can a nurse best evaluate the effectiveness of communication with a client?

Client feedback

What is the most important skill of the nurse leader?

Clinical care coordination

A client with chronic renal failure selects a scrambled egg for his breakfast. What action should the nurse take?

Commend the client for selecting a high biologic value protein.

The nurse notices that the Hispanic parents of a toddler who returns from surgery offer the child only the broth that comes on the clear liquid tray. Other liquids, including gelatin, popsicles, and juices, remain untouched. What explanation is most appropriate for this behavior? A. The belief is held that the "evil eye" enters the child if anything cold is ingested. B. After surgery the child probably has refused all foods except broth. C. Eating broth strengthens the child's innate energy called "chi." D. Hot remedies restore balance after surgery, which is considered a "cold" condition.

Common parental practices and health beliefs among Hispanic, Chinese, Filipino, and Arab cultures classify diseases, areas of the body, and illnesses as "hot" or "cold" and must be balanced to maintain health and prevent illness. The perception that surgery is a "cold" condition implies that only "hot" remedies, such as soup, should be used to restore the healthy balance within the body, so (D) is the correct interpretation. (A, B, and C) are not correct interpretations of the noted behavior. "Chi" is a Chinese belief that an innate energy enters and leaves the body via certain locations and pathways and maintains health. The "evil eye," or "mal ojo," is believed by many cultures to be related to the balance of health and illness but is unrelated to dietary practice. Correct Answer: D

A child is being treated with oral ampicillin for otitis media. What should be included in the discharge instructions that the nurse provides to the parents of the client?

Complete the entire course of antibiotic therapy.

The nurse notices that the mother of a 9-year-old Vietnamese child always looks at the floor when she talks to the nurse. What action should the nurse take?

Continue asking the mother questions about the child.

When conducting an admission assessment, the nurse should ask the client about the use of complimentary healing practices. Which statement is accurate regarding the use of these practices? A. Complimentary healing practices interfere with the efficacy of the medical model of treatment. B. Conventional medications are likely to interact with folk remedies and cause adverse effects. C. Many complimentary healing practices can be used in conjunction with conventional practices. D. Conventional medical practices will ultimately replace the use of complimentary healing practices.

Conventional approaches to health care can be depersonalizing and often fail to take into consideration all aspects of an individual, including body, mind, and spirit. Often complimentary healing practices can be used in conjunction with conventional medical practices (C), rather than interfering (A) with conventional practices, causing adverse effects (B), or replacing conventional medical care (D). Correct Answer: C

A hospitalized male client is receiving nasogastric tube feedings via a small-bore tube and a continuous pump infusion. He reports that he had a bad bout of severe coughing a few minutes ago, but feels fine now. What action is best for the nurse to take? A. Record the coughing incident. No further action is required at this time. B. Stop the feeding, explain to the family why it is being stopped, and notify the healthcare provider. C. After clearing the tube with 30 ml of air, check the pH of fluid withdrawn from the tube. D. Inject 30 ml of air into the tube while auscultating the epigastrium for gurgling.

Coughing, vomiting, and suctioning can precipitate displacement of the tip of the small bore feeding tube upward into the esophagus, placing the client at increased risk for aspiration. Checking the sample of fluid withdrawn from the tube (after clearing the tube with 30 ml of air) for acidic (stomach) or alkaline (intestine) values is a more sensitive method for these tubes, and the nurse should assess tube placement in this way prior to taking any other action (C). (A and B) are not indicated. The auscultating method (D) has been found to be unreliable for small-bore feeding tubes. Correct Answer: C

A Sub-Saharan African widowed immigrant woman lives with her deceased husband's brother and his family, which includes the brother-in-law's children and the widow's adult children. Each family member speaks fluent English. Surgery was recommended for the client. What is the best plan to obtain consent for surgery for this client? A. Obtain an interpreter to explain the procedure to the client. B. Encourage the client to make her own decision regarding surgery. C. Ask the family members to provide a clarification of the surgeon's explanation to the client. D. Tell the surgeon that the brother-in-law will decide after explanation of the proposed surgery is provided to him and the widow.

Customary law in some rural sub-Saharan countries encompasses wife inheritance and polygamy; the widow becomes the inherited wife of her husband's brother. In those rural areas women live in a patriarchal family where decisions are made by men. Most likely, the brother-in-law will make the decision for his inherited wife, so (D) provides the surgeon with culturally sensitive information. (A) all family members speak fluent English therefore there is no need for translation. It is culturally insensitive to encourage the woman to go against her wishes and her cultural worldview, as in (B). Family members are more likely to misinterpret medical information (C). Correct Answer: D

A client with acute hemorrhagic anemia is to receive four units of packed RBCs (red blood cells) as rapidly as possible. Which intervention is most important for the nurse to implement? A) Obtain the pre-transfusion hemoglobin level B) Prime the tubing and prepare a blood pump set-up C) Monitor vital signs q15 mintues for the first hour D) Ensure the accuracy of the blood type match

D) Ensure the accuracy of the blood type match

A male client tells the nurse that he does not know where he is or what year it is. What data should the nurse document that is most accurate? A) demonstrate loss of remote memory. B) exhibits expressive dysphasia C) has a diminished attention span D) is disoriented to place and time

D) is disoriented to place and time

During the initial morning assessment, a male client denies dysuria but reports that his urine appears dark amber. Which intervention should the nurse implement? A. Provide additional coffee on the client's breakfast tray. B. Exchange the client's grape juice for cranberry juice. C. Bring the client additional fruit at mid-morning. D. Encourage additional oral intake of juices and water.

Dark amber urine is characteristic of fluid volume deficit, and the client should be encouraged to increase fluid intake (D). Caffeine, however, is a diuretic (A), and may worsen the fluid volume deficit. Any type of juice will be beneficial (B), since the client is not dysuric, a sign of an urinary tract infection. The client needs to restore fluid volume more than solid foods (C). Correct Answer: D

Which intrinsic factor is associated with the fall of an older adult?

Deconditioning

During a visit to the outpatient clinic, the nurse assesses a client with severe osteoarthritis using a goniometer. Which finding should the nurse expect to measure?

Degree of flexion and extension of the client's knee joint.

An African-American grandmother tells the nurse that her 4-year-old grandson is suffering with "miseries." Based on this statement, which focused assessment should the nurse conduct? A. Inquire about the source and type of pain. B. Examine the nose for congestion and discharge. C. Take vital signs for temperature elevation. D. Explore the abdominal area for distension.

Different cultural groups often have their own terms for health conditions. African-American clients may refer to pain as "the miseries. " Based on understanding this term, the nurse should conduct a focused assessment on the source and type of pain (A). (B, C, and D) are important, but do not focus on "miseries" (pain). Correct Answer: A

At the beginning of the shift, the nurse assesses a client who is admitted from the post-anesthesia care unit (PACU). When should the nurse document the client's findings? A. A the beginning, middle, and end of the shift. B. After client priorities are identified for the development of the nursing care plan. C. At the end of the shift so full attention can be given to the client's needs. D. Immediately after the assessments are completed.

Documentation should occur immediately after any component of the nursing process, so assessments should be entered in the client's medical record as readily as findings are obtained (D). (A, B, and C) do not address the concepts of legal recommendations for information management and informatics. Correct Answer: D

The nurse is teaching a client proper use of an inhaler. When should the client administer the inhaler-delivered medication to demonstrate correct use of the inhaler?

During the inhalation

A nurse is reviewing a client's plan of care. What is the determining factor in the revision of the plan?

Effectiveness of the interventions

When caring for a client who is receiving enteral feedings, the nurse should take which measure to prevent aspiration?

Elevate the head of the bed between 30 and 45 degrees.

To prevent thrombophlebitis in the immediate postoperative period, which action is most important for a nurse to include in the client's plan of care?

Encourage early mobility

A client becomes anxious after being scheduled for a colostomy. What is the most effective way for the nurse to help the client?

Encourage the client to express feelings.

Which professional standard does the nurse feel is most important for critical thinking?

Evaluation criteria

Which assessment data would provide the most accurate determination of proper placement of a nasogastric tube?

Examining a chest x-ray obtained after the tubing was inserted.

A registered nurse instructs a nursing student to use knowledge and experience to choose proper strategies to use to care for clients. Which critical-thinking skill does the registered nurse refer to?

Explanation

Which critical thinking skill in nursing practice requires the nurse to possess knowledge and experience for choosing care strategies for clients?

Explanation

The nurse notices that the mother of a 9-year-old Vietnamese child always looks at the floor when she talks to the nurse. What action should the nurse take? A. Talk directly to the child instead of the mother. B. Continue asking the mother questions about the child. C. Ask another nurse to interview the mother now. D. Tell the mother politely to look at you when answering.

Eye contact is a culturally-influenced form of non-verbal communication. In some non-Western cultures, such as the Vietnamese culture, a client or family member may avoid eye contact as a form of respect, so the nurse should continue to ask the mother questions about the child (B). (A, C, and D) are not indicated. Correct Answer: B

A client with a leg prosthesis and a history of syncopal episodes is being admitted to the hospital. When formulating the plan of care for this client, the nurse should include that the client is at risk for what?

Falls

Which statement defines the term family resiliency?

Family resiliency is the ability of the family to cope with stressors.

When evaluating a client's plan of care, the nurse determines that a desired outcome was not achieved. Which action will the nurse implement first? A. Establish a new nursing diagnosis. B. Note which actions were not implemented. C. Add additional nursing orders to the plan. D. Collaborate with the healthcare provider to make changes.

First, the nurse reviews which actions in the original plan were not implemented (B) in order to determine why the original plan did not produce the desired outcome. Appropriate revisions can then be made, which may include revising the expected outcome, or identifying a new nursing diagnosis (A). (C) may be needed if the nursing actions were unsuccessful, or were unable to be implemented. (D) other members of the healthcare team may be necessary to collaborate changes once the nurse determines why the original plan did not produce the desired outcome. Correct Answer: B

The nurse is administering medications through a nasogastric tube (NGT) which is connected to suction. After ensuring correct tube placement, what action should the nurse take next?

Flush the tube with water.

A client with chronic renal failure selects a scrambled egg for his breakfast. What action should the nurse take? A. Commend the client for selecting a high biologic value protein. B. Remind the client that protein in the diet should be avoided. C. Suggest that the client also select orange juice, to promote absorption. D. Encourage the client to attend classes on dietary management of CRF.

Foods such as eggs and milk (A) are high biologic proteins which are allowed because they are complete proteins and supply the essential amino acids that are necessary for growth and cell repair. Although a low-protein diet is followed (B), some protein is essential. Orange juice is rich in potassium, and should not be encouraged (C). The client has made a good diet choice, so (D) is not necessary. Correct Answer: A

An elderly male client who is unresponsive following a cerebral vascular accident (CVA) is receiving bolus enteral feedings though a gastrostomy tube. What is the best client position for administration of the bolus tube feedings?

Fowler's.

An elderly male client who suffered a cerebral vascular accident is receiving tube feedings via a gastrostomy tube. The nurse knows that the best position for this client during administration of the feedings is

Fowler's.

The nurse is completing a mental assessment for a client who is demonstrating slow thought processes, personality changes, and emotional lability. Which area of the brain controls these neuro-cognitive functions?

Frontal lobe

Which theory proposes that older adults experience a shift from a materialistic to cosmic view of the world?

Gerotranscendence theory

A client who is in hospice care complains of increasing amounts of pain. The healthcare provider prescribes an analgesic every four hours as needed. Which action should the nurse implement?

Give an around-the-clock schedule for administration of analgesics.

A client is in the radiology department at 0900 when the prescription levofloxacin (Levaquin) 500 mg IV q24h is scheduled to be administered. The client returns to the unit at 1300. What is the best intervention for the nurse to implement?

Give the missed dose at 1300 and change the schedule to administer daily at 1300.

Which action is most important for the nurse to implement when donning sterile gloves? A. Maintain thumb at a ninety degree angle. B. Hold hands with fingers down while gloving. C. Keep gloved hands above the elbows. D. Put the glove on the dominant hand first.

Gloved hands held below waist level are considered unsterile (C). (A and B) are not essential to maintaining asepsis. While it may be helpful to put the glove on the dominant hand first, it is not necessary to ensure asepsis (D). Correct Answer: C

A nurse is teaching continuing care assistants about ways to prevent the spread of infection. It would be appropriate for the nurse to emphasize that the cycle of the infectious process must be broken, which is accomplished primarily through what?

Hand washing before and after providing client care

Which statement is true about the nursing model "team nursing"?

Hierarchical communication exists from charge nurse to charge nurse, charge nurse to team leader, and team leader to team members.

The nurse notices that the Hispanic parents of a toddler who returns from surgery offer the child only the broth that comes on the clear liquid tray. Other liquids, including gelatin, popsicles, and juices, remain untouched. What explanation is most appropriate for this behavior?

Hot remedies restore balance after surgery, which is considered a "cold" condition.

The nurse is instructing a client with high cholesterol about diet and life style modification. What comment from the client indicates that the teaching has been effective?

I will limit my intake of beef to 4 ounces per week.

A client who is in the advanced stages of illness asks the nurse to contact pastoral services for support. According to the Macmillan model, what is the best nursing intervention in this situation?

Immediately involve pastoral services while caring for the client.

An older client who is a resident in a long term care facility has been bedridden for a week. Which finding should the nurse identify as a client risk factor for pressure ulcers? A. Generalized dry skin. B. Localized dry skin on lower extremities. C. Red flush over entire skin surface. D. Rashes in the axillary, groin, and skin fold regions.

Immobility, constant with bed clothing, and excessive heat and moisture in areas where air flow is limited contributes to bacterial and fungal growth, which increases the risk for rashes (D), skin breakdown, and the development of pressure ulcers. (A, B, and C) do not address the concepts of inflammation and tissue integrity. Correct Answer: D

Which nursing process involves delegation and verbal discussion with the healthcare team?

Implementation

The nurse is caring for a client who is receiving 24-hour total parenteral nutrition (TPN) via a central line at 54 ml/hr. When initially assessing the client, the nurse notes that the TPN solution has run out and the next TPN solution is not available. What immediate action should the nurse take?

Infuse 10 percent dextrose and water at 54 ml/hr.

A client is receiving a cephalosporin antibiotic IV and complains of pain and irritation at the infusion site. The nurse observes erythema, swelling, and a red streak along the vessel above the IV access site. Which action should the nurse take at this time?

Initiate an alternate site for the IV infusion of the medication.

An African-American grandmother tells the nurse that her 4-year-old grandson is suffering with "miseries." Based on this statement, which focused assessment should the nurse conduct?

Inquire about the source and type of pain

Three days following surgery, a male client observes his colostomy for the first time. He becomes quite upset and tells the nurse that it is much bigger than he expected. What is the best response by the nurse?

Instruct the client that the stoma will become smaller when the initial swelling diminishes.

Which skill in critical thinking requires to be orderly in data collection?

Interpretation

The nurse assigns a UAP to obtain vital signs from a very anxious client. What instructions should the nurse give the UAP? A. Remain calm with the client and record abnormal results in the chart. B. Notify the medication nurse immediately if the pulse or blood pressure is low. C. Report the results of the vital signs to the nurse. D. Reassure the client that the vital signs are normal.

Interpretation of vital signs is the responsibility of the nurse, so the UAP should report vital sign measurements to the nurse (C). (A, B, and D) require the UAP to interpret the vital signs, which is beyond the scope of the UAP's authority. Correct Answer: C

A client tells the nurse, "I am so worried about the results of the biopsy they took today." The nurse overhears the nursing assistant reply, "Don't worry. I'm sure everything will come out all right." What does the nurse conclude about the nursing assistant's answer?

It gives false reassurance.

At the time of the first dressing change, the client refuses to look at her mastectomy incision. The nurse tells the client that the incision is healing well, but the client refuses to talk about it. What would be an appropriate response to this client's silence?

It is OK if you don't want to talk about your surgery. I will be available when you are ready.

An adult male client with a history of hypertension tells the nurse that he is tired of taking antihypertensive medications and is going to try spiritual meditation instead. What should be the nurse's first response?

It is important that you continue your medication while learning to meditate.

Which action is most important for the nurse to implement when donning sterile gloves?

Keep gloved hands above the elbows.

During the admission interview, which technique is most efficient for the nurse to use when obtaining information about signs and symptoms of a client's primary health problem? A. Restatement of responses. B. Open-ended questions. C. Closed-ended questions. D. Problem-seeking responses.

Lay descriptions of health problems can be vague and nonspecific. To efficiently obtain specific information, the nurse should use closed-ended questions (C) that focus on common signs and symptoms about a client's health problem. (A, B, and D) are used when therapeutically interacting and should be used after specific information is obtained from the client. Correct answer: C

The nurse is instructing a client with high cholesterol about diet and life style modification. What comment from the client indicates that the teaching has been effective? A. "If I exercise at least two times weekly for one hour, I will lower my cholesterol." B. "I need to avoid eating proteins, including red meat." C. "I will limit my intake of beef to 4 ounces per week." D. "My blood level of low density lipoproteins needs to increase."

Limiting saturated fat from animal food sources to no more than 4 ounces per week (C) is an important diet modification for lowering cholesterol. To be effective in reducing cholesterol, the client should exercise 30 minutes per day, or at least 4 to 6 times per week (A). Red meat and all proteins do not need to be eliminated (B) to lower cholesterol, but should be restricted to lean cuts of red meat and smaller portions (2-ounce servings). The low density lipoproteins (D) need to decrease rather than increase. Correct Answer: C

Examination of a client complaining of itching on his right arm reveals a rash made up of multiple flat areas of redness ranging from pinpoint to 0.5 cm in diameter. How should the nurse record this finding?

Localized red rash comprised of flat areas, pinpoint to 0.5 cm in diameter.

A client with cystic fibrosis asks why the percussion procedure is being performed. The nurse explains that the primary purpose of percussion is to do what?

Loosen pulmonary secretions

Examination of a client complaining of itching on his right arm reveals a rash made up of multiple flat areas of redness ranging from pinpoint to 0.5 cm in diameter. How should the nurse record this finding? A. Multiple vesicular areas surrounded by redness, ranging in size from 1 mm to 0.5 cm. B. Localized red rash comprised of flat areas, pinpoint to 0.5 cm in diameter. C. Several areas of red, papular lesions from pinpoint to 0.5 cm in size. D. Localized petechial areas, ranging in size from pinpoint to 0.5 cm in diameter.

Macules are localized flat skin discolorations less than 1 cm in diameter. However, when recording such a finding the nurse should describe the appearance (B) rather than simply naming the condition. (A) identifies vesicles -- fluid filled blisters -- an incorrect description given the symptoms listed. (C) identifies papules -- solid elevated lesions, again not correctly identifying the symptoms. (D) identifies petechiae -- pinpoint red to purple skin discolorations that do not itch, again an incorrect identification. Correct Answer: B

A client's infusion of normal saline infiltrated earlier today, and approximately 500 ml of saline infused into the subcutaneous tissue. The client is now complaining of excruciating arm pain and demanding "stronger pain medications." What initial action is most important for the nurse to take?

Measure the pulse volume and capillary refill distal to the infiltration.

A nurse is preparing to give medications through a nasogastric feeding tube. Which nursing action should prevent complications during administration? A. Mix each medication individually. B. Use sterile gloves for the procedure. C. Monitor vital signs before giving medications. D. Mix all medications together to facilitate administration.

Medications should be mixed separately (A) to prevent clumping. (B, C, and D) are not indicated. Correct Answer: A

The UAPs working on a chronic neuro unit ask the nurse to help them determine the safest way to transfer an elderly client with left-sided weakness from the bed to the chair. What method describes the correct transfer procedure for this client?

Move the chair parallel to the right side of the bed, and stand the client on the right foot.

A client is being admitted to a medical unit with a diagnosis of pulmonary tuberculosis. The nurse should assign the client to which type of room?

Negative-airflow room

The nurse is caring for a surgical client who develops a wound infection during hospitalization. How is this type of infection classified?

Nosocomial

During follow-up visits, the client's child reports to the nurse, "I tell my parent every day about what may happen if medications aren't taken as prescribed. Despite that, my parent does not take the medication regularly and is depressed." What can be inferred about the client's motivational level?

Not motivated

An elderly resident of a long-term care facility is no longer able to perform self-care and is becoming progressively weaker. The resident previously requested that no resuscitative efforts be performed, and the family requests hospice care. What action should the nurse implement first?

Notify the healthcare provider of the family's request.

A young mother of three children complains of increased anxiety during her annual physical exam. What information should the nurse obtain first?

Nutritional history.

The nurse observes that a male client has removed the covering from an ice pack applied to his knee. What action should the nurse take first?

Observe the appearance of the skin under the ice pack.

A client's infusion of normal saline infiltrated earlier today, and approximately 500 ml of saline infused into the subcutaneous tissue. The client is now complaining of excruciating arm pain and demanding stronger pain medications. What initial action is most important for the nurse to take? A. Ask about any past history of drug abuse or addiction. B. Measure the pulse volume and capillary refill distal to the infiltration. C. Compress the infiltrated tissue to measure the degree of edema. D. Evaluate the extent of ecchymosis over the forearm area.

Pain and diminished pulse volume (B) are signs of compartment syndrome, which can progress to complete loss of the peripheral pulse in the extremity. Compartment syndrome occurs when external pressure (usually from a cast), or internal pressure (usually from subcutaneous infused fluid), exceeds capillary perfusion pressure resulting in decreased blood flow to the extremity. (A) should not be pursued until physical causes of the pain are ruled out. (C) is of less priority than determining the effects of the edema on circulation and nerve function. Further assessment of the client's ecchymosis can be delayed until the signs of edema and compression that suggest compartment syndrome have been examined (D). Correct Answer: B

When monitoring a client 24 to 48 hours after abdominal surgery, the nurse should assess for which problem associated with anesthetic agents?

Paralytic ileus

A nurse takes into consideration that the key factor in accurately assessing how a client will cope with body image changes is what?

Perception of the change

The nurse receives information about a client through another nurse. The nurse then finds that information has some missing facts. Which critical thinking attitude would the nurse use to clarify the information after talking to the client directly?

Perseverance

A 73-year-old female client had a hemiarthroplasty of the left hip yesterday due to a fracture resulting from a fall. In reviewing hip precautions with the client, which instruction should the nurse include in this client's teaching plan?

Place a pillow between your knees while lying in bed to prevent hip dislocation.

A client being treated for influenza A (H1N1) is scheduled for a computed tomography (CT) scan. To ensure client and visitor safety during transport, the nurse should take which precaution?

Place a surgical mask on the client.

Three days following surgery, a male client observes his colostomy for the first time. He becomes quite upset and tells the nurse that it is much bigger than he expected. What is the best response by the nurse? A. Reassure the client that he will become accustomed to the stoma appearance in time. B. Instruct the client that the stoma will become smaller when the initial swelling diminishes. C. Offer to contact a member of the local ostomy support group to help him with his concerns. D. Encourage the client to handle the stoma equipment to gain confidence with the procedure.

Postoperative swelling causes enlargement of the stoma. The nurse can teach the client that the stoma will become smaller when the swelling is diminished (B). This will help reduce the client's anxiety and promote acceptance of the colostomy. (A) does not provide helpful teaching or support. (C) is a useful action, and may be taken after the nurse provides pertinent teaching. The client is not yet demonstrating readiness to learn colostomy care (D). Correct Answer: B

The nurse is providing restraint education to a group of nursing students. The nurse should include that it is inappropriate to use a restraint device to do what?

Prevent an adult client from getting up at night when there is insufficient staffing on the unit.

Which nurse collaborates directly with the client to establish and implement a basic plan of care after admission?

Primary nurse

The nurse is caring for a client who had a hip replacement 2 days prior. After removing a bedpan from under the client, what is a priority nursing intervention?

Provide perineal care

The nurse is performing nasotracheal suctioning. After suctioning the client's trachea for fifteen seconds, large amounts of thick yellow secretions return. What action should the nurse implement next?

Re-oxygenate the client before attempting to suction again.

An older adult in an acute care setting is experiencing emotional stress because of a recent surgery. Which intervention would be most appropriate for the client?

Reality orientation

The nurse observes an unlicensed assistive personnel (UAP) taking a client's blood pressure with a cuff that is too small, but the blood pressure reading obtained is within the client's usual range. What action is most important for the nurse to implement?

Reassess the client's blood pressure using a larger cuff.

Which nursing practice is associated with the self-regulation skill?

Reflecting on one's experience

Which nursing action would be considered a part of self-regulation in the decision-making process?

Reflecting on one's own experiences

Which action made by the client indicates that they are in the precontemplation stage of Transtheoretical Model of Change?

Refuses to think about changing

Nursing actions for an older adult should include health education and promotion of self-care. Which is most important when working with an older adult client?

Reinforcing the client's strengths and promoting reminiscing

In developing a plan of care for a client with dementia, the nurse should remember that confusion in the elderly A. is to be expected, and progresses with age. B. often follows relocation to new surroundings. C. is a result of irreversible brain pathology. D. can be prevented with adequate sleep.

Relocation (B) often results in confusion among elderly clients--moving is stressful for anyone. (A) is a stereotypical judgment. Stress in the elderly often manifests itself as confusion, so (C) is wrong. Adequate sleep is not a prevention (D) for confusion. Correct Answer: B

The nurse assigns a UAP to obtain vital signs from a very anxious client. What instructions should the nurse give the UAP?

Report the results of the vital signs to the nurse.

An unlicensed assistive personnel (UAP) places a client in a left lateral position prior to administering a soap suds enema. Which instruction should the nurse provide the UAP?

Reposition in a Sim's position with the client's weight on the anterior ilium.

A female client with a nasogastric tube attached to low suction states that she is nauseated. The nurse assesses that there has been no drainage through the nasogastric tube in the last two hours. What action should the nurse take first?

Reposition the client on her side

A female client asks the nurse to find someone who can translate into her native language her concerns about a treatment. Which action should the nurse take?

Request and document the name of the certified translator.

Which therapeutic communication technique involves using a coping strategy to help the nurse and client adjust to stress?

Sharing humor

Which developmental changes should be evaluated in girls around 12 years of age?

Skeletal growth

The nurse is evaluating client learning about a low-sodium diet. Selection of which meal would indicate to the nurse that this client understands the dietary restrictions?

Skim milk, turkey salad, roll, and vanilla ice cream.

The nurse is evaluating client learning about a low-sodium diet. Selection of which meal would indicate to the nurse that this client understands the dietary restrictions? A. Tossed salad, low-sodium dressing, bacon and tomato sandwich. B. New England clam chowder, no-salt crackers, fresh fruit salad. C. Skim milk, turkey salad, roll, and vanilla ice cream. D. Macaroni and cheese, diet Coke, a slice of cherry pie.

Skim milk, turkey, bread, and ice cream (C), while containing some sodium, are considered low-sodium foods. Bacon (A), canned soups (B), especially those with seafood, hard cheeses, macaroni, and most diet drinks (D) are very high in sodium. Correct Answer: C

The home healthcare nurse visits a client who has two grandchildren living in the household. The client's adult child is a single-parent who is in prison serving a 15-year sentence. The children accompany the grandparent on 2-hour contact visits on weekends as often as possible. Which term does the nurse use to define this family form?

Skip-generation family

A nurse is reviewing how a hyperglycemic client's blood glucose can be lowered. The nurse recalls that the chemical that buffers the client's excessive acetoacetic acid is what?

Sodium bicarbonate

The nurse is performing nasotracheal suctioning. After suctioning the client's trachea for fifteen seconds, large amounts of thick yellow secretions return. What action should the nurse implement next? A. Encourage the client to cough to help loosen secretions. B. Advise the client to increase the intake of oral fluids. C. Rotate the suction catheter to obtain any remaining secretions. D. Re-oxygenate the client before attempting to suction again.

Suctioning should not be continued for longer than ten to fifteen seconds, since the client's oxygenation is compromised during this time (D). (A, B, and C) may be performed after the client is re-oxygenated and additional suctioning is performed. Correct Answer: D

How should the nurse prevent footdrop in a client with a leg cast?

Support the foot with 90 degrees of flexion.

The nurse is caring for a client who is receiving 24-hour total parenteral nutrition (TPN) via a central line at 54 ml/hr. When initially assessing the client, the nurse notes that the TPN solution has run out and the next TPN solution is not available. What immediate action should the nurse take? A. Infuse normal saline at a keep vein open rate. B. Discontinue the IV and flush the port with heparin. C. Infuse 10 percent dextrose and water at 54 ml/hr. D. Obtain a stat blood glucose level and notify the healthcare provider.

TPN is discontinued gradually to allow the client to adjust to decreased levels of glucose. Administering 10% dextrose in water at the prescribed rate (C) will keep the client from experiencing hypoglycemia until the next TPN solution is available. The client could experience a hypoglycemic reaction if the current level of glucose (A) is not maintained or if the TPN is discontinued abruptly (B). There is no reason to obtain a stat blood glucose level (D) and the healthcare provider cannot do anything about this situation. Correct Answer: C

Refusing to follow the prescribed treatment regimen, a client plans to leave the hospital against medical advice. What is it important for the nurse to inform the client of?

That the client must accept full responsibility for possible undesirable outcomes

The nurse is administering medications through a nasogastric tube (NGT) which is connected to suction. After ensuring correct tube placement, what action should the nurse take next? A. Clamp the tube for 20 minutes. B. Flush the tube with water. C. Administer the medications as prescribed. D. Crush the tablets and dissolve in sterile water.

The NGT should be flushed before, after and in between each medication administered (B). Once all medications are administered, the NGT should be clamped for 20 minutes (A). (C and D) may be implemented only after the tubing has been flushed. Correct Answer: B

The nurse prepares a 1,000 ml IV of 5% dextrose and water to be infused over 8 hours. The infusion set delivers 10 drops per milliliter. The nurse should regulate the IV to administer approximately how many drops per minute? A. 80 B. 8 C. 21 D. 25

The accepted formula for figuring drops per minute is: amount to be infused in one hour × drop factor/time for infusion (min)= drops per minute. Using this formula: 1,000/8 hours = 125 ml/ hour 125 × 10 (drip factor) = 1,250 drops in one hour. 1,250/ 60 (number of minutes in one hour) = 20.8 or 21 gtt/min (C). Correct Answer: C

The nurse at the well baby clinic is assessing the gross motor skills of a five-month-old infant. Which finding is a cause for concern?

The baby has a head lag when pulled to sit.

When assisting an 82-year-old client to ambulate, it is important for the nurse to realize that the center of gravity for an elderly person is the A. Arms. B. Upper torso. C. Head. D. Feet.

The center of gravity for adults is the hips. However, as the person grows older, a stooped posture is common because of the changes from osteoporosis and normal bone degeneration, and the knees, hips, and elbows flex. This stooped posture results in the upper torso (B) becoming the center of gravity for older persons. Although (A) is a part, or an extension of the upper torso, this is not the best and most complete answer. Correct Answer: B

A community healthcare nurse is conducting a survey about homeless children in the community. Which finding helps the nurse distinguish absolute homelessness from relative homelessness?

The children do not have a physical shelter and may sleep outdoors or in vehicles.

A male client tells the nurse that he does not know where he is or what year it is. What data should the nurse document that is most accurate? A. demonstrates loss of remote memory. B. exhibits expressive dysphasia. C. has a diminished attention span. D. is disoriented to place and time.

The client is exhibiting disorientation (D). (A) refers to memory of the distant past. The client is able to express himself without difficulty (B), and does not demonstrate a diminished attention span (C). Correct Answer: D

During a home visit, the nurse finds that a healthy elderly person is actively practicing laughing therapy to maintain good health without pressure or insistence from family members. What does the nurse infer from these findings?

The client is intrinsically motivated.

The nurse is teaching a client proper use of an inhaler. When should the client administer the inhaler-delivered medication to demonstrate correct use of the inhaler? A. Immediately after exhalation. B. During the inhalation. C. At the end of three inhalations. D. Immediately after inhalation.

The client should be instructed to deliver the medication during the last part of inhalation (B). After the medication is delivered, the client should remove the mouthpiece, keeping his/her lips closed and breath held for several seconds to allow for distribution of the medication. The client should not deliver the dose as stated in (A or D), and should deliver no more than two inhalations at a time (C). Correct Answer: B

An elderly male client who is unresponsive following a cerebral vascular accident (CVA) is receiving bolus enteral feedings though a gastrostomy tube. What is the best client position for administration of the bolus tube feedings? A. Prone. B. Fowler's. C. Sims'. D. Supine.

The client should be positioned in a semi-sitting (Fowler's) (B) position during feeding to decrease the occurrence of aspiration. A gastrostomy tube, known as a PEG tube, due to placement by a percutaneous endoscopic gastrostomy procedure, is inserted directly into the stomach through an incision in the abdomen for long-term administration of nutrition and hydration in the debilitated client. In (A and/or C), the client is placed on the abdomen, an unsafe position for feeding. Placing the client in (D) increases the risk of aspiration. Correct Answer: B

The nurse witnesses the signature of a client who has signed an informed consent. Which statement best explains this nursing responsibility?

The client voluntarily signed the form.

The nurse creates a plan of care for a client with a risk of infection. Which is the most desirable expected outcome for the client?

The client will be free of signs and symptoms of infection by discharge.

The nurse is teaching a client with numerous allergies how to avoid allergens. Which instruction should be included in this teaching plan? A. Avoid any types of sprays, powders, and perfumes. B. Wearing a mask while cleaning will not help to avoid allergens. C. Purchase any type of clothing, but be sure it is washed before wearing it. D. Pollen count is related to hay fever, not to allergens.

The client with allergies should be instructed to reduce any exposure to pollen, dust, fumes, odors, sprays, powders, and perfumes (A). The client should be encouraged to wear a mask when working around dust or pollen (B). Clients with allergies should avoid any clothing that causes itching; washing clothes will not prevent an allergic reaction to some fabrics (C). Pollen count is related to allergens (D), and the client should be instructed to stay indoors when the pollen count is high. Correct Answer: A

A 73-year-old female client had a hemiarthroplasty of the left hip yesterday due to a fracture resulting from a fall. In reviewing hip precautions with the client, which instruction should the nurse include in this client's teaching plan? A. "In 8 weeks you will be able to bend at the waist to reach items on the floor." B. "Place a pillow between your knees while lying in bed to prevent hip dislocation." C. "It is safe to use a walker to get out of bed, but you need assistance when walking." D. "Take pain medication 30 minutes after your physical therapy sessions."

The client's affected hip joint following a hemiarthroplasty (partial hip replacement) is at risk of dislocation for 6 months to a year following the procedure. Hip precautions to prevent dislocation include placing a pillow between the knees to maintain abduction of the hips (B). Clients should be instructed to avoid bending at the waist (A), to seek assistance for both standing and walking until they are stable on a walker or cane (C), and to take pain medication 20 to 30 minutes prior to physical therapy sessions, rather than waiting until the pain level is high after their therapy. Correct Answer: B

A client who wakes up after a surgery spits out the oral airway placed during the recovery from anesthesia. What does this behavior indicate to the nurse?

The client's gag reflex has returned.

The nurse observes that a male client has removed the covering from an ice pack applied to his knee. What action should the nurse take first? A. Observe the appearance of the skin under the ice pack. B. Instruct the client regarding the need for the covering. C. Reapply the covering after filling with fresh ice. D. Ask the client how long the ice was applied to the skin.

The first action taken by the nurse should be to assess the skin for any possible thermal injury (A). If no injury to the skin has occurred, the nurse can take the other actions (B, C, and D) as needed. Correct Answer: A

An elderly client who requires frequent monitoring fell and fractured a hip. Which nurse is at greatest risk for a malpractice judgment? A. A nurse who worked the 7 to 3 shift at the hospital and wrote poor nursing notes. B. The nurse assigned to care for the client who was at lunch at the time of the fall. C. The nurse who transferred the client to the chair when the fall occurred. D. The charge nurse who completed rounds 30 minutes before the fall occurred.

The four elements of malpractice are: breach of duty owed, failure to adhere to the recognized standard of care, direct causation of injury, and evidence of actual injury. The hip fracture is the actual injury and the standard of care was "frequent monitoring." (C) implies that duty was owed and the injury occurred while the nurse was in charge of the client's care. There is no evidence of negligence in (A, B, and D). Correct Answer: C

The nurse is completing a mental assessment for a client who is demonstrating slow thought processes, personality changes, and emotional lability. Which area of the brain controls these neuro-cognitive functions? A. Thalamus. B. Hypothalamus. C. Frontal lobe. D. Parietal lobe.

The frontal lobe (C) of the cerebrum controls higher mental activities, such as memory, intellect, language, emotions, and personality. (A) is an afferent relay center in the brain that directs impulses to the cerebral cortex. (B) regulates body temperature, appetite, maintains a wakeful state, and links higher centers with the autonomic nervous and endocrine systems, such as the pituitary. (D) is the location of sensory and motor functions. Correct Answer: C

During a visit to the outpatient clinic, the nurse assesses a client with severe osteoarthritis using a goniometer. Which finding should the nurse expect to measure? A. Adequate venous blood flow to the lower extremities. B. Estimated amount of body fat by an underarm skinfold. C. Degree of flexion and extension of the client's knee joint. D. Change in the circumference of the joint in centimeters.

The goniometer is a two-piece ruler that is jointed in the middle with a protractor-type measuring device that is placed over a joint as the individual extends or flexes the joint to measure the degrees of flexion and extension on the protractor (C). A doppler is used to measure blood flow (A). Calipers are used to measure body fat (B). A tape measure is used to measure circumference of body parts (D). Correct Answer: C

A female client with a nasogastric tube attached to low suction states that she is nauseated. The nurse assesses that there has been no drainage through the nasogastric tube in the last two hours. What action should the nurse take first? A. Irrigate the nasogastric tube with sterile normal saline. B. Reposition the client on her side. C. Advance the nasogastric tube an additional five centimeters. D. Administer an intravenous antiemetic prescribed for PRN use.

The immediate priority is to determine if the tube is functioning correctly, which would then relieve the client's nausea. The least invasive intervention, (B), should be attempted first, followed by (A and C), unless either of these interventions is contraindicated. If these measures are unsuccessful, the client may require an antiemetic (D). Correct Answer: B

An unlicensed assistive personnel (UAP) places a client in a left lateral position prior to administering a soap suds enema. Which instruction should the nurse provide the UAP? A. Position the client on the right side of the bed in reverse Trendelenburg. B. Fill the enema container with 1000 ml of warm water and 5 ml of castile soap. C. Reposition in a Sim's position with the client's weight on the anterior ilium. D. Raise the side rails on both sides of the bed and elevate the bed to waist level.

The left sided Sims' position allows the enema solution to follow the anatomical course of the intestines and allows the best overall results, so the UAP should reposition the client in the Sims' position, which distributes the client's weight to the anterior ilium (C). (A) is inaccurate. (B and D) should be implemented once the client is positioned. Correct Answer: C

A client who is in hospice care complains of increasing amounts of pain. The healthcare provider prescribes an analgesic every four hours as needed. Which action should the nurse implement? A. Give an around-the-clock schedule for administration of analgesics. B. Administer analgesic medication as needed when the pain is severe. C. Provide medication to keep the client sedated and unaware of stimuli. D. Offer a medication-free period so that the client can do daily activities.

The most effective management of pain is achieved using an around-the-clock schedule that provides analgesic medications on a regular basis (A) and in a timely manner. Analgesics are less effective if pain persists until it is severe, so an analgesic medication should be administered before the client's pain peaks (B). Providing comfort is a priority for the client who is dying, but sedation that impairs the client's ability to interact and experience the time before life ends should be minimized (C). Offering a medication-free period allows the serum drug level to fall, which is not an effective method to manage chronic pain (D). Correct Answer: A

The nurse observes an unlicensed assistive personnel (UAP) taking a client's blood pressure with a cuff that is too small, but the blood pressure reading obtained is within the client's usual range. What action is most important for the nurse to implement? A. Tell the UAP to use a larger cuff at the next scheduled assessment. B. Reassess the client's blood pressure using a larger cuff. C. Have the unit educator review this procedure with the UAPs. D. Teach the UAP the correct technique for assessing blood pressure.

The most important action is to ensure that an accurate BP reading is obtained. The nurse should reassess the BP with the correct size cuff (B). Reassessment should not be postponed (A). Though (C and D) are likely indicated, these actions do not have the priority of (B). Correct Answer: B

An obese male client discusses with the nurse his plans to begin a long-term weight loss regimen. In addition to dietary changes, he plans to begin an intensive aerobic exercise program 3 to 4 times a week and to take stress management classes. After praising the client for his decision, which instruction is most important for the nurse to provide? A. "Be sure to have a complete physical examination before beginning your planned exercise program." B. "Be careful that the exercise program doesn't simply add to your stress level, making you want to eat more." C. "Increased exercise helps to reduce stress, so you may not need to spend money on a stress management class." D. "Make sure to monitor your weight loss regularly to provide a sense of accomplishment and motivation."

The most important teaching is (A), so that the client will not begin a dangerous level of exercise when he is not sufficiently fit. This might result in chest pain, a heart attack, or stroke. (B, C, and D) are important instructions, but are of less priority than (A). Correct Answer: A

A nurse is taking the vital signs of a client who has just been admitted to the healthcare facility. Which intervention by the nurse provides greater client satisfaction?

The nurse adjusts the bed and asks if the client is comfortable.

An elderly resident of a long-term care facility is no longer able to perform self-care and is becoming progressively weaker. The resident previously requested that no resuscitative efforts be performed, and the family requests hospice care. What action should the nurse implement first? A. Reaffirm the client's desire for no resuscitative efforts. B. Transfer the client to a hospice inpatient facility. C. Prepare the family for the client's impending death. D. Notify the healthcare provider of the family's request.

The nurse should first communicate with the healthcare provider (D). Hospice care is provided for clients with a limited life expectancy, which must be identified by the healthcare provider. (A) is not necessary at this time. Once the healthcare provider supports the transfer to hospice care, the nurse can collaborate with the hospice staff and healthcare provider to determine when (B and C) should be implemented. Correct Answer: D

The nurse witnesses the signature of a client who has signed an informed consent. Which statement best explains this nursing responsibility? A. The client voluntarily signed the form. B. The client fully understands the procedure. C. The client agrees with the procedure to be done. D. The client authorizes continued treatment.

The nurse signs the consent form to witness that the client voluntarily signs the consent (A), that the client's signature is authentic, and that the client is otherwise competent to give consent. It is the healthcare provider's responsibility to ensure the client fully understands the procedure (B). The nurse's signature does not indicate (C or D). Correct Answer: A

An elderly client who requires frequent monitoring fell and fractured a hip. Which nurse is at greatest risk for a malpractice judgment?

The nurse who transferred the client to the chair when the fall occurred.

When assessing a client with wrist restraints, the nurse observes that the fingers on the right hand are blue. What action should the nurse implement first? A. Loosen the right wrist restraint. B. Apply a pulse oximeter to the right hand. C. Compare hand color bilaterally. D. Palpate the right radial pulse.

The priority nursing action is to restore circulation by loosening the restraint (A), because blue fingers (cyanosis) indicates decreased circulation. (C and D) are also important nursing interventions, but do not have the priority of (A). Pulse oximetry (B) measures the saturation of hemoglobin with oxygen and is not indicated in situations where the cyanosis is related to mechanical compression (the restraints). Correct Answer: A

An adult male client with a history of hypertension tells the nurse that he is tired of taking antihypertensive medications and is going to try spiritual meditation instead. What should be the nurse's first response? A. "It is important that you continue your medication while learning to meditate." B. "Spiritual meditation requires a time commitment of 15 to 20 minutes daily." C. "Obtain your healthcare provider's permission before starting meditation." D. "Complementary therapy and western medicine can be effective for you."

The prolonged practice of meditation may lead to a reduced need for antihypertensive medications. However, the medications must be continued (A) while the physiologic response to meditation is monitored. (B) is not as important as continuing the medication. The healthcare provider should be informed, but permission is not required to meditate (C). Although it is true that this complimentary therapy might be effective (D), it is essential that the client continue with antihypertensive medications until the effect of meditation can be measured. Correct Answer: A

A client who has been NPO for 3 days is receiving an infusion of D5W 0.45 normal saline (NS) with potassium chloride (KCl) 20 mEq at 83 mL/hour. The client's eight-hour urine output is 400 mL blood urea nitrogen (BUN) is 15 mg/dL, lungs are clear bilaterally, serum glucose is 120 mg/dL, and the serum potassium is 3.7 mEq/L. Which action is most important for the nurse to implement? A. Notify healthcare provider and request to change the IV infusion to hypertonic D 10W. B. Decrease in the infusion rate of the current IV and report to the healthcare provider. C. Document in the medical record that these normal findings are expected outcomes. D. Obtain potassium chloride 20 mEq in anticipation of a prescription to add to present IV.

The results are all within normal range (C). No changes are needed (A, B, and D). Correct Answer: C

While instructing a male client's wife in the performance of passive range-of-motion exercises to his contracted shoulder, the nurse observes that she is holding his arm above and below the elbow. What nursing action should the nurse implement? A. Acknowledge that she is supporting the arm correctly. B. Encourage her to keep the joint covered to maintain warmth. C. Reinforce the need to grip directly under the joint for better support. D. Instruct her to grip directly over the joint for better motion.

The wife is performing the passive ROM correctly, therefore the nurse should acknowledge this fact (A). The joint that is being exercised should be uncovered (B) while the rest of the body should remain covered for warmth and privacy. (C and D) do not provide adequate support to the joint while still allowing for joint movement. Correct Answer: A

A male client being discharged with a prescription for the bronchodilator theophylline tells the nurse that he understands he is to take three doses of the medication each day. Since, at the time of discharge, timed-release capsules are not available, which dosing schedule should the nurse advise the client to follow? A. 9 a.m., 1 p.m., and 5 p.m. B. 8 a.m., 4 p.m., and midnight. C. Before breakfast, before lunch and before dinner. D. With breakfast, with lunch, and with dinner.

Theophylline should be administered on a regular around-the-clock schedule (B) to provide the best bronchodilating effect and reduce the potential for adverse effects. (A, C, and D) do not provide around-the-clock dosing. Food may alter absorption of the medication (D). Correct Answer: B

An elderly client with a fractured left hip is on strict bedrest. Which nursing measure is essential to the client's nursing care? A. Massage any reddened areas for at least five minutes. B. Encourage active range of motion exercises on extremities. C. Position the client laterally, prone, and dorsally in sequence. D. Gently lift the client when moving into a desired position.

To avoid shearing forces when repositioning, the client should be lifted gently across a surface (D). Reddened areas should not be massaged (A) since this may increase the damage to already traumatized skin. To control pain and muscle spasms, active range of motion (B) may be limited on the affected leg. The position described in (C) is contraindicated for a client with a fractured left hip. Correct Answer: D

A client is in the radiology department at 0900 when the prescription levofloxacin (Levaquin) 500 mg IV q24h is scheduled to be administered. The client returns to the unit at 1300. What is the best intervention for the nurse to implement? A. Contact the healthcare provider and complete a medication variance form. B. Administer the Levaquin at 1300 and resume the 0900 schedule in the morning. C. Notify the charge nurse and complete an incident report to explain the missed dose. D. Give the missed dose at 1300 and change the schedule to administer daily at 1300.

To ensure that a therapeutic level of medication is maintained, the nurse should administer the missed dose as soon as possible, and revise the administration schedule accordingly to prevent dangerously increasing the level of the medication in the bloodstream (D). The nurse should document the reason for the late dose, but (A and C) are not warranted. (B) could result in increased blood levels of the drug. Correct Answer: D

What is the role of a case manager in a healthcare organization?

To follow up with the client after discharge

The nurse providing care for a client with a diagnosis of neutropenia reviews isolation procedures with the client's spouse. The nurse determines that the teaching was effective when the spouse states that protective environment isolation helps prevent the spread of infection in which direction?

To the client from outside sources

Which activity would the nurse explain can be performed by infants of aged 6 to 8 months?

Transferring objects from hand to hand

Which intervention reflects the nurse's approach of "family as a context"?

Trying to meet the client's comfort

Which site is best for the nurse to obtain a urinalysis specimen for a critical care client with an indwelling urine catheter?

Tubing luer-lok port

Which nutritional assessment data should the nurse collect to best reflect total muscle mass in an adolescent? A. Height in inches or centimeters. B. Weight in kilograms or pounds. C. Triceps skin fold thickness. D. Upper arm circumference.

Upper arm circumference (D) is an indirect measure of muscle mass. (A and B) do not distinguish between fat (adipose) and muscularity. (C) is a measure of body fat. Correct Answer: D

Which intervention is most important for the nurse to implement for a male client who is experiencing urinary retention? A. Apply a condom catheter. B. Apply a skin protectant. C. Encourage increased fluid intake. D. Assess for bladder distention.

Urinary retention is the inability to void all urine collected in the bladder, which leads to uncomfortable bladder distention (D). (A and B) are useful actions to protect the skin of a client with urinary incontinence. (C) may worsen the bladder distention. Correct Answer: D

What type of functional health pattern would the nurse explain describes values and goals?

Value-belief pattern

What is the most important reason for starting intravenous infusions in the upper extremities rather than the lower extremities of adults? A. It is more difficult to find a superficial vein in the feet and ankles. B. A decreased flow rate could result in the formation of a thrombosis. C. A cannulated extremity is more difficult to move when the leg or foot is used. D. Veins are located deep in the feet and ankles, resulting in a more painful procedure.

Venous return is usually better in the upper extremities. Cannulation of the veins in the lower extremities increases the risk of thrombus formation (B) which, if dislodged, could be life-threatening. Superficial veins are often very easy (A) to find in the feet and legs. Handling a leg or foot with an IV (C) is probably not any more difficult than handling an arm or hand. Even if the nurse did believe moving a cannulated leg was more difficult, this is not the most important reason for using the upper extremities. Pain (D) is not a consideration. Correct Answer: B

A client who is 5' 5" tall and weighs 200 pounds is scheduled for surgery the next day. What question is most important for the nurse to include during the preoperative assessment? A. "What is your daily calorie consumption?" B. "What vitamin and mineral supplements do you take?: C. "Do you feel that you are overweight?" D. "Will a clear liquid diet be okay after surgery?"

Vitamin and mineral supplements (B) may impact medications used during the operative period. (A and C) are appropriate questions for long-term dietary counseling. The nature of the surgery and anesthesia will determine the need for a clear liquid diet (D), rather than the client's preference. Correct Answer: B

During the daily nursing assessment, a client begins to cry and states that the majority of family and friends have stopped calling and visiting. What action should the nurse take? A. Listen and show interest as the client expresses these feelings. B. Reinforce that this behavior means they were not true friends C. Ask the healthcare provider for a psychiatric consult. D. Continue with the assessment and tell the client not to worry.

When a client begins to cry and express feelings, a therapeutic nursing intervention is to listen and show interest as the client expresses feelings (A). (B) is not therapeutic option and the nurse does not know the dynamics of their relationships. (C) is not indicated at this time. (D) is non-therapeutic and offers false hope. Correct Answer: A

A resident in a skilled nursing facility for short-term rehabilitation after a hip replacement tells the nurse, "I don't want any more blood taken for those useless tests." Which narrative documentation should the nurse enter in the client's medical record? A. Healthcare provider notified of failure to collect specimens for prescribed blood studies. B. Blood specimens not collected because client no longer wants blood tests performed. C. Healthcare provider notified of client's refusal to have blood specimens collected for testing. D. Client irritable, uncooperative, and refuses to have blood collected. Healthcare provider notified.

When a client refuses a treatment, the exact words of the client regarding the client's refusal of care should be documented in a narrative format (C). (A, B, and D) do not address the concepts of informatics and legal issues. Correct Answer: C

Twenty minutes after beginning a heat application, the client states that the heating pad no longer feels warm enough. What is the best response by the nurse?

Your blood vessels are becoming dilated and removing the heat from the site.

The practical nurse (PN) is evaluating a client for an increased need for B12 after undergoing a subtotal antrectomy. Which foods should the ON recommend that the client include in the diet? a. cheese, eggs, and fish b. lentils, dried fruits, and soy beans c. potatoes mushrooms, and yogurt d. nuts, brown rice, and sunflower seeds

a. cheese, eggs, and fish these foods are rich in B12

A client who arrives at the urgent care clinic reporting diarrhea, ever, abdominal pain and nausea and tells the practical nurse the symptoms began after eating a can of beans. What is the likely cause of this type of foodbourne illness? a. clostridium botulinum b. salmonella typhi c. listeria monocytogenes d. campylobacter jejuni

a. clostridium botulinum

The practical nurse interviews a client who had a total colectomy weeks ago. The client claims to be confident in the management of the new colostomy, but is having problems managing the odor. What suggestions should the PN provide the client to help decrease odor in the colostomy bag? (SATA) a. eat foods containing yogurt b. chew mint flavored gum c. drink a glass of buttermilk d. eat sprigs of parsley e. consume raw vegetables

a. eat foods containing yogurt c. drink a glass of buttermilk d. eat sprigs of parsley

The practical nurse is reviewing the dietary recommendations with the client who had a partial gastrectomy and experienced dumping syndrome. Which statement by the client indicates that the recommendations were understood? a. fluids should be limited to eight ounces with meals b. rice should be avoided from the client's diet c. sugar-free gelatin should be used with caution d. meat should consist of no more than 4 ounces or 120 ml/day

a. fluids should be limited to eight ounces with meals

A client who is lactose intolerant asks the practical nurse (PN) for suggestions about different food sources that are enriched in calcium and vitamin D. Which foods should the PN suggest? a. fortified soy products b. whole wheat bread c. prune or cranberry juice d. red berries and pears

a. fortified soy products

The practical nurse (PN) is assisting an Orthodox Jewish client who has dementia make weekly dinner choices in an assisted living community. Which main course selections should the PN suggest that are suitable for a kosher diet? a. lamb chops with mint jelly b. crab cakes with cocktail sauce c. pork chops with cranberry relish d. steak with cream sauce

a. lamb chops with mint jelly

During a nutrition consultation for elevated cholesterol, the dietician recommends that the client replace saturated fats with monounsaturated or polyunsaturated fats. What explanation should the practical nurse reenforce with the client about this change in fat in the diet? a. lowers the amount of low density lipoprotein (LDL) in the blood b. lowers the amount of high density lipoprotein (HDL) in the blood c. contributes o raising cholesterol levels in the blood d. contributes o raising triglycerides levels in the blood

a. lowers the amount of low density lipoprotein (LDL) in the blood

A client develops a vitamin K deficiency due to intestinal malabsorption. What intervention should the practical nurse implement in the plan of care? a. monitor for signs of hematuria, melena, ecchymosis b. increase intake of leafy green vegetables c. drink orange juice with a prescribed daily iron supplement d. monitor for cardiac irregularities

a. monitor for signs of hematuria, melena, ecchymosis

An elderly client in an assisted living community is refusing treat during meal time. The practical nurse (PN) discovers the the client has several mouth ulcers. What type of food should the PN suggest to promote intake and healing ulcers? a. poached eggs or scrambled eggs b. hot sauce and broths c. peanut butter crackers d. ice cream or milk shakes

a. poached eggs or scrambled eggs bland or soft food with a significant amount of protein content is best

The practical nurse reviews the dietary treatment plan with a client who has ascites secondary to cirrhosis of the liver. Which information should the PN include when reviewing the dietary plan? a. restricted sodium intake b. increased protein intake c. restricted fluid intake d. increase potassium intake

a. restricted sodium intake

The practical nurse is reenforcing diet teaching for a client with a simple goiter. Which foods should the ON recommend that the client eliminate from the diet? a. turnips b. oranges c. milk d. fish

a. turnips turnips are considered a exogenous goitrogen, which are thyroid-inhibiting substances and should be avoided if the client has a goiter

The practical nurse (PN) reviews self-care with a male client who is newly diagnosed with cirrhosis. Which vitamin deficit due to the liver damage should the PN discuss with the client that increases his susceptibility of bleeding? a. vit K b. vit C c. vit E d. vit B

a. vit K clients with cirrhosis have a decrease in bile production which is necessary for fat emulsification and absorption of fat-soluble vitamin K and a deficit in vitamin K puts the client at risk for bleeding due to a decrease in hepatic synthesis of clotting factors, such as prothrombin

The practical nurse (PN) is evaluating the client's knowledge of low-sodium menu selections. Which food items listed by the client confirm appropriate choices? (SATA) a. white rice with steamed vegetables b. cottage cheese with sliced tomatoes c. canned vegetable soup with crackers d. turkey bacon with scrambled eggs e. chicken with soy sauce and brown rice

a. white rice with steamed vegetables b. cottage cheese with sliced tomatoes

The healthcare provider prescribes a protein supplement for an older client who is recovering from surgery. What information should the practical nurse provide the family about the value of the supplement? a. an increase of protein supplies fuel for energy in the client b. additional protein promotes tissue healing postoperatively c. protein supplements stimulate the client's appetite d. increased protein satiates craving for carbohydrates

b. additional protein promotes tissue healing postoperatively incised tissue needs additional protein to help build and repair cells

The practical nurse is reviewing a list of foods with the family that are recommended for an older client who recently had a total colectomy and colostomy. Which food item should the PN discuss about including in the client's postoperative diet? a. carbonated beverages b. chicen noodle soup c. boiled cabbage d. bean burritos

b. chicken noodle soup

The practical nurse (PN) is caring for a group of clients who are on a high protein diet. Which client's condition is most likely to experience the most therapeutic response with this diet? a. client with cardiovascular disease b. client recovering from illness or surgery c. client who is experiencing a stressful life event d. client who exercises regularly after sporadic activity

b. client recovering from illness or surgery

The practical nurse (PN) is discharging a new mother from he post partum unit. Which statement made by the client during discharge indicates that the new mother needs reenforcement of the breastfeeding instructions? a. increase caloric intake by a minimum of 500 calories t aid with milk production b. drink one glass of wine while breast feeding to help with milk "let down" c. drink fluids liberally between 2 to 3 liters/day to help with milk production d. spicy foods should be avoided to reduce the incidence of causing distress for the baby

b. drink one glass of wine while breast feeding to help with milk "let down"

The practical nurse (PN) is reviewing a meal plan of foods rich in riboflavin for primigravida client, Which foods should the PN suggest that the client eat at least 2 servings daily? a. fresh fruits b. enriched cereals c. white potatoes d. white rice

b. enriched cereals

The practical nurse is reinforcing teaching to a client who is on a low residue diet. Which foods should the PN recommend that the client avoid? a. baked potato b. hard salami c. cottage cheese d. scribbled eggs

b. hard salami

The practical nurse is visiting a client at home who asks if commercially prepared food products can age or spoil. Which foods should the PN explain have the likelihood of aging and creating a byproduct that is not healthy? a. jar of peaches b. jar of peanut butter c. canned chicken d. canned salmon

b. jar of peanut butter

The practical nurse (PN) is caring for an older adult with malnutrition. What factor is most likely contributing to this older client's nutritional status? a. increased need for vitamins and minerals b. loss of teeth or poorly fitting dentures c. a decline in kidney function d. snacks eaten ask day without a complete meal

b. loss of teeth or poorly fitting dentures

The practical nurse is caring for a client with continuous feeding through a nasogastric tube at a continuous care rehabilitation community. Which position should the PN place the client to prevent aspiration? a. trendelenburg b. semi-fowlers c. sims d. supine

b. semi-fowlers

An older client receives a prescription for psylium (Metamucil) for constipation. What information should the practical nurse (PN) provide the client about taking this supplement? a. take medication in the evening before bedtime to reduce fullness b. take supplement with 240 ml of liquid and follow with a second glass c. the supplement with how beverages to help dissolve the powder completely d. mix supplement with pudding or applesauce to help the taste and texture

b. take supplement with 240 ml of liquid and follow with a second glass psylium (Metamucil) is a bulk laxative and should be taken with full glass followed by another glass of fluid to be effective and decrease the chance of impaction

The practical nurse (PN) is reinforcing the teaching of diet and weight loss with a client who has the goal of losing 25 lbs. What is the most common problem that the PN should recognize about the accuracy of client reporting about daily food intake? a. over reporting pf food intake b. under reporting of food intake c. unawareness of the amount of food eaten d. unwillingness to change eating habits

b. under reporting of food intake a client is more likely to under report what has been eaten due to the embarrassment or inability to comply with the weight loss goal

A Chinese male client who moves into an assisted living community is concerned that the community will not be able to meet his dietary preferences. What should the practical nurse include in the diet for this client based on his cultural group? a. dairy products b. vegetables c. pork and beef d. cakes and pies

b. vegetables a Chinese diet includes generous amounts of rice and vegetables which are often included in every meal

The practical nurse (PN) suspects that a male client is altering his own diabetic journals to please his healthcare provider. Which laboratory test should the PN review to evaluate the client's compliance with self management for type 1 diabetes mellitus? a. oral glucose tolerance test (GTT) b. 24-hour urine analysis c. hemoglobin A1c d. fasting cholesterol

c. hemoglobin A1c the A1c measures the average blood glucose level over the past 3-months, including the past 24 hour period, and should be used to compare with the client's diabetic journal

The home health practical cure (PN) is caring for a client with a stage III pressure ulcer. Which food group that contains zinc should be added to the client's diet to aide in wound healing? a. cheese and eggs b. green apples and berries c. meats and shellfish d. complex carbohydrates

c. meats and shellfish

The practical nurse (PN) is caring for a client who was recently diagnosed with type 2 diabetes mellitus. What information is most important for the PN to reinforce with the client about life-style changes? a. daily fingerstick glucose monitoring b. regular exercise program c. portion-controlled, heart healthy diet selections d. compliance with oral hypoglycemic medications

c. portion-controlled, heart healthy diet selections

The practical nurse is preparing a client about a stool guaiac test. Which food should the PN tell the client to avoid 3 days prior to collecting the specimen for the test? a. shellfish b. pasta c. raw broccoli d. peanut butter

c. raw broccoli

A client is admitted with right upper quadrant pain after eating. What should the practical nurse ask the client about the meal's content that precipitated the pain? a. protein source b. refined carbohydrates c. saturated fat content d. fresh, raw vegetables

c. saturated fat content

The practical nurse reviews the clients medical record that indicates the client has pernicious anemia. Which vitamin deficit is associated with this type of anemia? a. vit D b. vit B6 c. vit B12 d. vit C

c. vit B12 pernicious anemia - inability to absorb vit B12 due to a deficit in hydrochloric acid

The practical nurse (PN) receives four new admissions in an assissted living community. After reviewing the clients' medical histories, which client should the PN evaluate for a high caloric diet? a. a client with osteoporosis b. a client with type 2 diabetes mellitus c. a client with rheumatoid arthritis d. a clean with bacterial pneumonia

d. a clean with bacterial pneumonia a client with an active infection requires an increase in caloric intake to combat the infection

The practical nurse (PN) is reviewing guidelines to manage stress incontinence with a female client. Which dietary change should the PN emphasize that will benefit the client? a. omit smoked and salted foods b. limit fluids to less than 2 liters a day c. reduce intake of proceeded foods d. avoid alcohol and caffeine

d. avoid alcohol and caffeine

The practical nurse (PN) is reviewing the diet with a client with acute cholecystitis. Which food selection demonstrates to the PN that the client understands the dietary recommendations? a. chicken pot pie and buttered rolls b. sautéed ground beef with mashed potatoes c. friend round steak and macaroni with cheese d. baked fish and tomatoes with lettuce

d. baked fish and tomatoes with lettuce a client with acute cholecystitis should decrease dietary fat in foods, so baked foods are the best choice

The practical nurse (PN) stresses to a female client the importance of osteoporosis prevention. Which foods should the PN recommend the client to include in her diet? a. apples b. cheddar cheese c. lima beans d. canned tuna

d. canned tuna

The practical nurse is reinforcing dietary instructions to the parents of a 6-month-old baby boy diagnosed with phenylketonuria (PKU). Which foods should the PN direct the parents to eliminate in the infant's diet? a. grapes and berries b. green vegetables c. cereals and breads d. chicken and fish

d. chicken and fish

The practical nurse is discussing nutritional options with a female client who had mild hypertension. Which information should the PN recommend the client change in her daily diet? a. avoid green leafy vegetables b. choose foods with simple sugars c. limit foods high in fiber d. decrease intake of canned foods

d. decrease intake of canned foods

The practical nurse (PN) is caring for a client who is receiving total parenteral nutrition (TPN). Which metabolic change should the PN monitor the client for during administration of TPN? a. hypercalcemia b. hypernatremia c. hyperkalemia d. hyperglycemia

d. hyperglycemia

The healthcare provider prescribed an oral glucose tolerance test for a male client with polyuria and polydipsia. The client is instructed to be NPO after midnight. What should the client be advised to drink with his morning medications? a. any clear liquids b. tea without sugar c. coffee without sugar d. water without flavor

d. water without flavor

A postoperative client will need to perform daily dressing changes after discharge. Which outcome statement best demonstrates the client's readiness to manage his wound care after discharge? The client

demonstrates the wound care procedure correctly.

After completing an assessment and determining that a client has a problem, which action should the nurse perform next?

determine the etiology of the problem

The healthcare provider prescribes 1,000 ml of Ringer's Lactate with 30 Units of Pitocin to run in over 4 hours for a client who has just delivered a 10 pound infant by cesarean section. The tubing has been changed to a 20 gtt/ml administration set. The nurse plans to set the flow rate at how many gtt/min? A. 42 gtt/min. B. 83 gtt/min. C. 125 gtt/min. D. 250 gtt/min.

gtt/min = 20gtts/ml X 1000 ml/4hrs X 1 hr/60 min Correct Answer: B

A male client tells the nurse that he does not know where he is or what year it is. What data should the nurse document that is most accurate?

is disoriented to place and time.

In developing a plan of care for a client with dementia, the nurse should remember that confusion in the elderly

often follows relocation to new surroundings.

Which nutritional assessment data should the nurse collect to best reflect total muscle mass in an adolescent?

upper arm circumference


Ensembles d'études connexes